Partial sums of primesWhy do primes dislike dividing the sum of all the preceding primes?Partial sums of multiplicative functionsAre all primes in a PAP-3?Uniform distribution of digits of 1/pPrimes $p$ for which $pk+1$ is prime for small $k$ (or approximating Sophie Germain)Sums of primes that are themselves primeNew proofs of Euclid's theorem of the infinitude of primes?For $k>3$ does there exist an odd prime $q_k$ such that $p_k=2^kq_k+1$ is prime and $p_k$ divides $a_k=dfrac3^2^k-1+12$?Two equivalent statements about primesPermutations $piin S_n$ with $p_k+p_pi(k)+1$ prime for all $k=1,ldots,n$Can such a triple of palindromatic numbers exist?

Partial sums of primes


Why do primes dislike dividing the sum of all the preceding primes?Partial sums of multiplicative functionsAre all primes in a PAP-3?Uniform distribution of digits of 1/pPrimes $p$ for which $pk+1$ is prime for small $k$ (or approximating Sophie Germain)Sums of primes that are themselves primeNew proofs of Euclid's theorem of the infinitude of primes?For $k>3$ does there exist an odd prime $q_k$ such that $p_k=2^kq_k+1$ is prime and $p_k$ divides $a_k=dfrac3^2^k-1+12$?Two equivalent statements about primesPermutations $piin S_n$ with $p_k+p_pi(k)+1$ prime for all $k=1,ldots,n$Can such a triple of palindromatic numbers exist?













5












$begingroup$


$2+3+5+7+11+13...$ is clearly the sum of the primes.



Now I consider partial sums such:



$2+3+5+7+11=28$ which is divisible by $7$



My question is:



are there infinitely many partial sums such that:



$p_1+p_2+p_3+...+p_k+p_k+1=m*p_k?$ with $m$ some positive integer? With Pari/gp apparently up to 10^10 there are only two examples $7$=$p_k$ and $8263=p_k$. Heuristically do you think that infinitely many such partial sums should exist? Note: 7 and 8263 are both primes belonging to primes on the left side of the triangle formed by listing successively the prime numbers in a triangular grid. See https://oeis.org/A078721
Note in both cases $2+3+5+7=17$ is prime and $2+3+5+...+p_1036=3974497$ is prime. I note that $17$ and $3974497$ are primes of the form $4s+1$, whereas $p_4=7$ and $p_1036=8263$ are primes of the form $6s+1$.
$7$ and $8263$ are primes such that starting from the right, the odd positioned digits are prime and the even positioned digits are composite. But also $5$ and $8243$ which are the previous primes have this property. No other prime of this type found below $10^12$
I noticed that 7! has 4 digits where 4 is a palindrome. 8263! has 28782 digits where 28782 is a palindrome.










share|cite|improve this question











$endgroup$







  • 6




    $begingroup$
    Strongly related: mathoverflow.net/questions/120511/…. Also crossposted on MSE: math.stackexchange.com/questions/3161810/23571113 (please don't do this anymore).
    $endgroup$
    – Alex M.
    Mar 25 at 22:31







  • 3




    $begingroup$
    Seven edits in the last 12 hours.
    $endgroup$
    – Gerry Myerson
    Mar 26 at 21:11






  • 2




    $begingroup$
    Now up to Version 13.
    $endgroup$
    – Gerry Myerson
    Mar 27 at 21:35






  • 3




    $begingroup$
    I find these frequent edits go against the purpose of this forum. If you want to record frequent observations on a daily basis (whether they are significant or not), start a blog. You have asked a main question and gotten a reasonable answer; now move on. The numerology associated with the problem does not belong here. Next week, if you find a third prime satisfying the relations, you can report that here. Gerhard "Know When To Fold 'Em" Paseman, 2019.03.28.
    $endgroup$
    – Gerhard Paseman
    Mar 28 at 18:54






  • 2




    $begingroup$
    Version 16. Please, homunc, give it a rest.
    $endgroup$
    – Gerry Myerson
    Mar 28 at 21:30















5












$begingroup$


$2+3+5+7+11+13...$ is clearly the sum of the primes.



Now I consider partial sums such:



$2+3+5+7+11=28$ which is divisible by $7$



My question is:



are there infinitely many partial sums such that:



$p_1+p_2+p_3+...+p_k+p_k+1=m*p_k?$ with $m$ some positive integer? With Pari/gp apparently up to 10^10 there are only two examples $7$=$p_k$ and $8263=p_k$. Heuristically do you think that infinitely many such partial sums should exist? Note: 7 and 8263 are both primes belonging to primes on the left side of the triangle formed by listing successively the prime numbers in a triangular grid. See https://oeis.org/A078721
Note in both cases $2+3+5+7=17$ is prime and $2+3+5+...+p_1036=3974497$ is prime. I note that $17$ and $3974497$ are primes of the form $4s+1$, whereas $p_4=7$ and $p_1036=8263$ are primes of the form $6s+1$.
$7$ and $8263$ are primes such that starting from the right, the odd positioned digits are prime and the even positioned digits are composite. But also $5$ and $8243$ which are the previous primes have this property. No other prime of this type found below $10^12$
I noticed that 7! has 4 digits where 4 is a palindrome. 8263! has 28782 digits where 28782 is a palindrome.










share|cite|improve this question











$endgroup$







  • 6




    $begingroup$
    Strongly related: mathoverflow.net/questions/120511/…. Also crossposted on MSE: math.stackexchange.com/questions/3161810/23571113 (please don't do this anymore).
    $endgroup$
    – Alex M.
    Mar 25 at 22:31







  • 3




    $begingroup$
    Seven edits in the last 12 hours.
    $endgroup$
    – Gerry Myerson
    Mar 26 at 21:11






  • 2




    $begingroup$
    Now up to Version 13.
    $endgroup$
    – Gerry Myerson
    Mar 27 at 21:35






  • 3




    $begingroup$
    I find these frequent edits go against the purpose of this forum. If you want to record frequent observations on a daily basis (whether they are significant or not), start a blog. You have asked a main question and gotten a reasonable answer; now move on. The numerology associated with the problem does not belong here. Next week, if you find a third prime satisfying the relations, you can report that here. Gerhard "Know When To Fold 'Em" Paseman, 2019.03.28.
    $endgroup$
    – Gerhard Paseman
    Mar 28 at 18:54






  • 2




    $begingroup$
    Version 16. Please, homunc, give it a rest.
    $endgroup$
    – Gerry Myerson
    Mar 28 at 21:30













5












5








5


1



$begingroup$


$2+3+5+7+11+13...$ is clearly the sum of the primes.



Now I consider partial sums such:



$2+3+5+7+11=28$ which is divisible by $7$



My question is:



are there infinitely many partial sums such that:



$p_1+p_2+p_3+...+p_k+p_k+1=m*p_k?$ with $m$ some positive integer? With Pari/gp apparently up to 10^10 there are only two examples $7$=$p_k$ and $8263=p_k$. Heuristically do you think that infinitely many such partial sums should exist? Note: 7 and 8263 are both primes belonging to primes on the left side of the triangle formed by listing successively the prime numbers in a triangular grid. See https://oeis.org/A078721
Note in both cases $2+3+5+7=17$ is prime and $2+3+5+...+p_1036=3974497$ is prime. I note that $17$ and $3974497$ are primes of the form $4s+1$, whereas $p_4=7$ and $p_1036=8263$ are primes of the form $6s+1$.
$7$ and $8263$ are primes such that starting from the right, the odd positioned digits are prime and the even positioned digits are composite. But also $5$ and $8243$ which are the previous primes have this property. No other prime of this type found below $10^12$
I noticed that 7! has 4 digits where 4 is a palindrome. 8263! has 28782 digits where 28782 is a palindrome.










share|cite|improve this question











$endgroup$




$2+3+5+7+11+13...$ is clearly the sum of the primes.



Now I consider partial sums such:



$2+3+5+7+11=28$ which is divisible by $7$



My question is:



are there infinitely many partial sums such that:



$p_1+p_2+p_3+...+p_k+p_k+1=m*p_k?$ with $m$ some positive integer? With Pari/gp apparently up to 10^10 there are only two examples $7$=$p_k$ and $8263=p_k$. Heuristically do you think that infinitely many such partial sums should exist? Note: 7 and 8263 are both primes belonging to primes on the left side of the triangle formed by listing successively the prime numbers in a triangular grid. See https://oeis.org/A078721
Note in both cases $2+3+5+7=17$ is prime and $2+3+5+...+p_1036=3974497$ is prime. I note that $17$ and $3974497$ are primes of the form $4s+1$, whereas $p_4=7$ and $p_1036=8263$ are primes of the form $6s+1$.
$7$ and $8263$ are primes such that starting from the right, the odd positioned digits are prime and the even positioned digits are composite. But also $5$ and $8243$ which are the previous primes have this property. No other prime of this type found below $10^12$
I noticed that 7! has 4 digits where 4 is a palindrome. 8263! has 28782 digits where 28782 is a palindrome.







nt.number-theory prime-numbers






share|cite|improve this question















share|cite|improve this question













share|cite|improve this question




share|cite|improve this question








edited Mar 28 at 18:09







Enzo Creti

















asked Mar 25 at 15:56









Enzo CretiEnzo Creti

1




1







  • 6




    $begingroup$
    Strongly related: mathoverflow.net/questions/120511/…. Also crossposted on MSE: math.stackexchange.com/questions/3161810/23571113 (please don't do this anymore).
    $endgroup$
    – Alex M.
    Mar 25 at 22:31







  • 3




    $begingroup$
    Seven edits in the last 12 hours.
    $endgroup$
    – Gerry Myerson
    Mar 26 at 21:11






  • 2




    $begingroup$
    Now up to Version 13.
    $endgroup$
    – Gerry Myerson
    Mar 27 at 21:35






  • 3




    $begingroup$
    I find these frequent edits go against the purpose of this forum. If you want to record frequent observations on a daily basis (whether they are significant or not), start a blog. You have asked a main question and gotten a reasonable answer; now move on. The numerology associated with the problem does not belong here. Next week, if you find a third prime satisfying the relations, you can report that here. Gerhard "Know When To Fold 'Em" Paseman, 2019.03.28.
    $endgroup$
    – Gerhard Paseman
    Mar 28 at 18:54






  • 2




    $begingroup$
    Version 16. Please, homunc, give it a rest.
    $endgroup$
    – Gerry Myerson
    Mar 28 at 21:30












  • 6




    $begingroup$
    Strongly related: mathoverflow.net/questions/120511/…. Also crossposted on MSE: math.stackexchange.com/questions/3161810/23571113 (please don't do this anymore).
    $endgroup$
    – Alex M.
    Mar 25 at 22:31







  • 3




    $begingroup$
    Seven edits in the last 12 hours.
    $endgroup$
    – Gerry Myerson
    Mar 26 at 21:11






  • 2




    $begingroup$
    Now up to Version 13.
    $endgroup$
    – Gerry Myerson
    Mar 27 at 21:35






  • 3




    $begingroup$
    I find these frequent edits go against the purpose of this forum. If you want to record frequent observations on a daily basis (whether they are significant or not), start a blog. You have asked a main question and gotten a reasonable answer; now move on. The numerology associated with the problem does not belong here. Next week, if you find a third prime satisfying the relations, you can report that here. Gerhard "Know When To Fold 'Em" Paseman, 2019.03.28.
    $endgroup$
    – Gerhard Paseman
    Mar 28 at 18:54






  • 2




    $begingroup$
    Version 16. Please, homunc, give it a rest.
    $endgroup$
    – Gerry Myerson
    Mar 28 at 21:30







6




6




$begingroup$
Strongly related: mathoverflow.net/questions/120511/…. Also crossposted on MSE: math.stackexchange.com/questions/3161810/23571113 (please don't do this anymore).
$endgroup$
– Alex M.
Mar 25 at 22:31





$begingroup$
Strongly related: mathoverflow.net/questions/120511/…. Also crossposted on MSE: math.stackexchange.com/questions/3161810/23571113 (please don't do this anymore).
$endgroup$
– Alex M.
Mar 25 at 22:31





3




3




$begingroup$
Seven edits in the last 12 hours.
$endgroup$
– Gerry Myerson
Mar 26 at 21:11




$begingroup$
Seven edits in the last 12 hours.
$endgroup$
– Gerry Myerson
Mar 26 at 21:11




2




2




$begingroup$
Now up to Version 13.
$endgroup$
– Gerry Myerson
Mar 27 at 21:35




$begingroup$
Now up to Version 13.
$endgroup$
– Gerry Myerson
Mar 27 at 21:35




3




3




$begingroup$
I find these frequent edits go against the purpose of this forum. If you want to record frequent observations on a daily basis (whether they are significant or not), start a blog. You have asked a main question and gotten a reasonable answer; now move on. The numerology associated with the problem does not belong here. Next week, if you find a third prime satisfying the relations, you can report that here. Gerhard "Know When To Fold 'Em" Paseman, 2019.03.28.
$endgroup$
– Gerhard Paseman
Mar 28 at 18:54




$begingroup$
I find these frequent edits go against the purpose of this forum. If you want to record frequent observations on a daily basis (whether they are significant or not), start a blog. You have asked a main question and gotten a reasonable answer; now move on. The numerology associated with the problem does not belong here. Next week, if you find a third prime satisfying the relations, you can report that here. Gerhard "Know When To Fold 'Em" Paseman, 2019.03.28.
$endgroup$
– Gerhard Paseman
Mar 28 at 18:54




2




2




$begingroup$
Version 16. Please, homunc, give it a rest.
$endgroup$
– Gerry Myerson
Mar 28 at 21:30




$begingroup$
Version 16. Please, homunc, give it a rest.
$endgroup$
– Gerry Myerson
Mar 28 at 21:30










1 Answer
1






active

oldest

votes


















16












$begingroup$

You asked for a heuristic answer.



There is an heuristic argument that infinitely many such partial sums should exist. Consider $P(k)$, an heuristic estimate of the probability that the partial sum of the first $k+1$ primes would be divisible by $p_k$. Now $$p_k sim k log k$$ and if only random chance were involved, $$P(k) approx frac1p_k sim frac1k log k$$



In that case, the expected number of primes with the property you want would be something like
$$int_2^infty frac1x log x,dx$$
and that integral diverges to infinity.



The reason it seems so rare is that the rate of divergence is like $log(log x)$ and while that function goes to infinity, "nobody ever sees it do so."



On the other hand, proving that there an infinite number of such values of $k$ (in the same sense that Euclid's argument proves there is no last prime) is probably quite difficult. And if the conjecture that there are an infinite number of such values of $k$ turned out to be false, proving that some particular $k$ is the last one with this property would seem to be even harder.






share|cite|improve this answer











$endgroup$








  • 2




    $begingroup$
    Note this answer is essentially the same as David Speyer's in the question linked to in the comment by @Alex M. above.
    $endgroup$
    – Kimball
    Mar 25 at 23:30






  • 1




    $begingroup$
    "nobody ever sees it do so." - you made my day!
    $endgroup$
    – Wolfgang
    Mar 26 at 9:26













Your Answer








StackExchange.ready(function()
var channelOptions =
tags: "".split(" "),
id: "504"
;
initTagRenderer("".split(" "), "".split(" "), channelOptions);

StackExchange.using("externalEditor", function()
// Have to fire editor after snippets, if snippets enabled
if (StackExchange.settings.snippets.snippetsEnabled)
StackExchange.using("snippets", function()
createEditor();
);

else
createEditor();

);

function createEditor()
StackExchange.prepareEditor(
heartbeatType: 'answer',
autoActivateHeartbeat: false,
convertImagesToLinks: true,
noModals: true,
showLowRepImageUploadWarning: true,
reputationToPostImages: 10,
bindNavPrevention: true,
postfix: "",
imageUploader:
brandingHtml: "Powered by u003ca class="icon-imgur-white" href="https://imgur.com/"u003eu003c/au003e",
contentPolicyHtml: "User contributions licensed under u003ca href="https://creativecommons.org/licenses/by-sa/3.0/"u003ecc by-sa 3.0 with attribution requiredu003c/au003e u003ca href="https://stackoverflow.com/legal/content-policy"u003e(content policy)u003c/au003e",
allowUrls: true
,
noCode: true, onDemand: true,
discardSelector: ".discard-answer"
,immediatelyShowMarkdownHelp:true
);



);













draft saved

draft discarded


















StackExchange.ready(
function ()
StackExchange.openid.initPostLogin('.new-post-login', 'https%3a%2f%2fmathoverflow.net%2fquestions%2f326315%2fpartial-sums-of-primes%23new-answer', 'question_page');

);

Post as a guest















Required, but never shown

























1 Answer
1






active

oldest

votes








1 Answer
1






active

oldest

votes









active

oldest

votes






active

oldest

votes









16












$begingroup$

You asked for a heuristic answer.



There is an heuristic argument that infinitely many such partial sums should exist. Consider $P(k)$, an heuristic estimate of the probability that the partial sum of the first $k+1$ primes would be divisible by $p_k$. Now $$p_k sim k log k$$ and if only random chance were involved, $$P(k) approx frac1p_k sim frac1k log k$$



In that case, the expected number of primes with the property you want would be something like
$$int_2^infty frac1x log x,dx$$
and that integral diverges to infinity.



The reason it seems so rare is that the rate of divergence is like $log(log x)$ and while that function goes to infinity, "nobody ever sees it do so."



On the other hand, proving that there an infinite number of such values of $k$ (in the same sense that Euclid's argument proves there is no last prime) is probably quite difficult. And if the conjecture that there are an infinite number of such values of $k$ turned out to be false, proving that some particular $k$ is the last one with this property would seem to be even harder.






share|cite|improve this answer











$endgroup$








  • 2




    $begingroup$
    Note this answer is essentially the same as David Speyer's in the question linked to in the comment by @Alex M. above.
    $endgroup$
    – Kimball
    Mar 25 at 23:30






  • 1




    $begingroup$
    "nobody ever sees it do so." - you made my day!
    $endgroup$
    – Wolfgang
    Mar 26 at 9:26















16












$begingroup$

You asked for a heuristic answer.



There is an heuristic argument that infinitely many such partial sums should exist. Consider $P(k)$, an heuristic estimate of the probability that the partial sum of the first $k+1$ primes would be divisible by $p_k$. Now $$p_k sim k log k$$ and if only random chance were involved, $$P(k) approx frac1p_k sim frac1k log k$$



In that case, the expected number of primes with the property you want would be something like
$$int_2^infty frac1x log x,dx$$
and that integral diverges to infinity.



The reason it seems so rare is that the rate of divergence is like $log(log x)$ and while that function goes to infinity, "nobody ever sees it do so."



On the other hand, proving that there an infinite number of such values of $k$ (in the same sense that Euclid's argument proves there is no last prime) is probably quite difficult. And if the conjecture that there are an infinite number of such values of $k$ turned out to be false, proving that some particular $k$ is the last one with this property would seem to be even harder.






share|cite|improve this answer











$endgroup$








  • 2




    $begingroup$
    Note this answer is essentially the same as David Speyer's in the question linked to in the comment by @Alex M. above.
    $endgroup$
    – Kimball
    Mar 25 at 23:30






  • 1




    $begingroup$
    "nobody ever sees it do so." - you made my day!
    $endgroup$
    – Wolfgang
    Mar 26 at 9:26













16












16








16





$begingroup$

You asked for a heuristic answer.



There is an heuristic argument that infinitely many such partial sums should exist. Consider $P(k)$, an heuristic estimate of the probability that the partial sum of the first $k+1$ primes would be divisible by $p_k$. Now $$p_k sim k log k$$ and if only random chance were involved, $$P(k) approx frac1p_k sim frac1k log k$$



In that case, the expected number of primes with the property you want would be something like
$$int_2^infty frac1x log x,dx$$
and that integral diverges to infinity.



The reason it seems so rare is that the rate of divergence is like $log(log x)$ and while that function goes to infinity, "nobody ever sees it do so."



On the other hand, proving that there an infinite number of such values of $k$ (in the same sense that Euclid's argument proves there is no last prime) is probably quite difficult. And if the conjecture that there are an infinite number of such values of $k$ turned out to be false, proving that some particular $k$ is the last one with this property would seem to be even harder.






share|cite|improve this answer











$endgroup$



You asked for a heuristic answer.



There is an heuristic argument that infinitely many such partial sums should exist. Consider $P(k)$, an heuristic estimate of the probability that the partial sum of the first $k+1$ primes would be divisible by $p_k$. Now $$p_k sim k log k$$ and if only random chance were involved, $$P(k) approx frac1p_k sim frac1k log k$$



In that case, the expected number of primes with the property you want would be something like
$$int_2^infty frac1x log x,dx$$
and that integral diverges to infinity.



The reason it seems so rare is that the rate of divergence is like $log(log x)$ and while that function goes to infinity, "nobody ever sees it do so."



On the other hand, proving that there an infinite number of such values of $k$ (in the same sense that Euclid's argument proves there is no last prime) is probably quite difficult. And if the conjecture that there are an infinite number of such values of $k$ turned out to be false, proving that some particular $k$ is the last one with this property would seem to be even harder.







share|cite|improve this answer














share|cite|improve this answer



share|cite|improve this answer








edited Mar 25 at 22:43









Peter Taylor

1536 bronze badges




1536 bronze badges










answered Mar 25 at 16:42









Mark FischlerMark Fischler

1,0193 silver badges13 bronze badges




1,0193 silver badges13 bronze badges







  • 2




    $begingroup$
    Note this answer is essentially the same as David Speyer's in the question linked to in the comment by @Alex M. above.
    $endgroup$
    – Kimball
    Mar 25 at 23:30






  • 1




    $begingroup$
    "nobody ever sees it do so." - you made my day!
    $endgroup$
    – Wolfgang
    Mar 26 at 9:26












  • 2




    $begingroup$
    Note this answer is essentially the same as David Speyer's in the question linked to in the comment by @Alex M. above.
    $endgroup$
    – Kimball
    Mar 25 at 23:30






  • 1




    $begingroup$
    "nobody ever sees it do so." - you made my day!
    $endgroup$
    – Wolfgang
    Mar 26 at 9:26







2




2




$begingroup$
Note this answer is essentially the same as David Speyer's in the question linked to in the comment by @Alex M. above.
$endgroup$
– Kimball
Mar 25 at 23:30




$begingroup$
Note this answer is essentially the same as David Speyer's in the question linked to in the comment by @Alex M. above.
$endgroup$
– Kimball
Mar 25 at 23:30




1




1




$begingroup$
"nobody ever sees it do so." - you made my day!
$endgroup$
– Wolfgang
Mar 26 at 9:26




$begingroup$
"nobody ever sees it do so." - you made my day!
$endgroup$
– Wolfgang
Mar 26 at 9:26

















draft saved

draft discarded
















































Thanks for contributing an answer to MathOverflow!


  • Please be sure to answer the question. Provide details and share your research!

But avoid


  • Asking for help, clarification, or responding to other answers.

  • Making statements based on opinion; back them up with references or personal experience.

Use MathJax to format equations. MathJax reference.


To learn more, see our tips on writing great answers.




draft saved


draft discarded














StackExchange.ready(
function ()
StackExchange.openid.initPostLogin('.new-post-login', 'https%3a%2f%2fmathoverflow.net%2fquestions%2f326315%2fpartial-sums-of-primes%23new-answer', 'question_page');

);

Post as a guest















Required, but never shown





















































Required, but never shown














Required, but never shown












Required, but never shown







Required, but never shown

































Required, but never shown














Required, but never shown












Required, but never shown







Required, but never shown







Popular posts from this blog

Kamusi Yaliyomo Aina za kamusi | Muundo wa kamusi | Faida za kamusi | Dhima ya picha katika kamusi | Marejeo | Tazama pia | Viungo vya nje | UrambazajiKuhusu kamusiGo-SwahiliWiki-KamusiKamusi ya Kiswahili na Kiingerezakuihariri na kuongeza habari

Swift 4 - func physicsWorld not invoked on collision? The Next CEO of Stack OverflowHow to call Objective-C code from Swift#ifdef replacement in the Swift language@selector() in Swift?#pragma mark in Swift?Swift for loop: for index, element in array?dispatch_after - GCD in Swift?Swift Beta performance: sorting arraysSplit a String into an array in Swift?The use of Swift 3 @objc inference in Swift 4 mode is deprecated?How to optimize UITableViewCell, because my UITableView lags

Access current req object everywhere in Node.js ExpressWhy are global variables considered bad practice? (node.js)Using req & res across functionsHow do I get the path to the current script with Node.js?What is Node.js' Connect, Express and “middleware”?Node.js w/ express error handling in callbackHow to access the GET parameters after “?” in Express?Modify Node.js req object parametersAccess “app” variable inside of ExpressJS/ConnectJS middleware?Node.js Express app - request objectAngular Http Module considered middleware?Session variables in ExpressJSAdd properties to the req object in expressjs with Typescript